Chuyên đề bất đảng thức

11 1.5K 8
Chuyên đề bất đảng thức

Đang tải... (xem toàn văn)

Tài liệu hạn chế xem trước, để xem đầy đủ mời bạn chọn Tải xuống

Thông tin tài liệu

BAT ẹANG THệC SCHUR TRONG HèNH HOẽC Problem 5 The triangle ABC has unequal sides, centroid G, incenter I and orthocenter H. Show that angle GIH > 90o. Solution Let N be the midpoint of OH. Then IN = (IO + IH)/2, so IH = 2IN - IO (we use bold to represent vectors). G lies on the line OH with OG = OH/3 (the Euler line), so OG = 2GN and hence IG = (2IN + IO)/3. Hence IH.IG = (4IN2 - IO2)/3. We have the well-known results OI2 = R2 - 2rR (Euler's formula and IN = R/2 - r (Feuerbach's theorem - usually stated as "the incircle and the nine-point circle touch" - N is the center of the nine-point circle and R/2 is the radius of the nine-point circle). Hence IH.IG = (R2 - 4Rr + 4r2 - R2 + 2Rr)/3 = -2r(R - 2r)/3 < 0. If you are fluent with vector formulae for the triangle, the following solution by Mehul Srivastav is straightforward Use vectors origin O the circumcenter. Take the vector OA to be A etc. Then G = (A + B + C)/3, H = A + B + C (Euler line), I = (aA + bB + cC)/(a+b+c). The last formula is not so well-known, but is easy to verify. Check, for example, that b AI.AB = c AI.AC (for that it is convenient to relocate the origin to A). We have to show that (G-I).(H-I) < 0, or G.H + I2 - I.(G+H) < 0. Note that since the origin is the circumcenter we have B.C = R2cos2A = R2cos2A - R2sin2A = (R2 - a2/4) - a2/4 = R2 - a2/2. Similarly for C.A and A.B. Obviously A2 = B2 = C2 = R2. Hence 3G.H = A2 + B2 + C2 + 2A.B + 2B.C + 2C.A = 9R2 - (a2 + b2 + c2). We have I2 = (aA + bB + cC)2/(a+b+c)2 = ( (a2 + b2 + c2)R2 + 2ab(R2 - c2/2) + 2ac(R2 - b2/2) + 2bc(R2 - a2/2) )/ (a+b+c)2 = R2 - (abc2 + ab2c + a2bc)/(a+b+c)2 = R2 - abc/(a+b+c). (3/4)(a+b+c)I.(G+H) = (aA + bB + cC).(A + B + C) = (a+b+c)R2 + (a+b)A.B + (b+c)B.C + (c+a)C.A = 3(a+b+c)R2 - (c2(a+b) + a2(b+c) + b2(a+c))/2. So we wish to show that 3R2 - (a2+b2+c2)/3 + R2 -abc/(a+b+c) - 4R2 + (2/3)(a2b + b2a + . )/(a+b+c) < 0, or (a+b+c) (a2+b2+c2) + 3abc > 2(a2b + b2a + . ) or a3 + b3 + c3 - (a2b + b2a + . ) + 3abc > 0 or a(a-b)(a-c) + b(b-a)(b-c) + c(c- a)(c-b) > 0 (*). wlog a > b > c. So a(a-c) - b(b-c) > 0. Hence a(a-b)(a-c) + b(b-a)(b-c) > 0. Obviously c(c-a)(c-b) > 0. So (*) holds and he nce the result. BT NG THC SCHUR V PHNG PHP I BIN P,Q,R Vừ Thnh Vn Lp 11 Toỏn-Khi chuyờn THPT-HKH Hu *LI M U: Nh cỏc bn ó bit,bt ng thc Schur l mt bt ng thc mnh v cú nhiu ng dng,tuy nhiờn nú vn cũn khỏ xa l vi nhiu bn hc sinh THCS cng nh THPT.Qua bi vit ny,tụi mun cng cp thờm cho cỏc bn mt k thut s dng tt BDT Schur,ú l kt hp vi phng phỏp i bin . Trc ht tụi xin nhc li v bt ng thc Schur v phng phỏp i bin . I-BT NG THC SCHUR: Vi cỏc s thc dng a,b,c v bt kỡ ta luụn cú Hai trng hp quen thuc c s dng nhiu l k=1 v k=2: II-PHNG PHP I BIN P,Q,R: i vi mt s bi bt ng thc thun nht i xng cú cỏc bin khụng õm thỡ ta cú th i bin li nh sau: t Ta cú mt s ng thc sau: . Đặt Khi đó Có thể thấy ngay lợi ích của phương pháp này là mối ràng buộc giữa các biến p,q,r mà các biến a,b,c ban đầu không có như: Những kết quả trên đây chắc chắn là chưa đủ,các bạn có thể phát triển thêm nhiều đẳng thức,bất đẳng thức liên hệ giữa 3 biến p,q,r.Và điều quan trọng mà tôi muốn nói đến là từ bất đẳng thức và ,ta có: (từ ) (từ ) Tuy nhiên trong một số trường hợp thì có thể các đại lượng có thể nhận giá trị âm lẫn giá trị dương nên ta thường sử dụng . Có lẽ đến đây các bạn đã hiểu được phần nào về bất đẳng thức Schur và phương pháp đổi biến p,q,r.Sau đây là một số ví dụ minh họa,nhưng trước hết,các bạn hãy tập làm thử rồi xem đáp án sau: III-VÍ DỤ MINH HỌA 3.1:Bất đẳng thức Schur : Ví dụ 1 : Võ Thành Văn : Cho a,b,c là các số thực không âm.Chứng minh rằng: Lời giải: Đặt Áp dụng BDT Holder,ta có: Ta cần chứng minh: (đúng theo BDT Schur) Vậy ta có đpcm. Ví dụ 2 : APMO 2004 : Cho 3 số thực dương .Chứng minh rằng: Lời giải Lời giải 1:Khai triển bất đẳng thức trên,ta cần chứng minh: Ta có: (theo BDT Schur) Áp dụng các BDT trên,ta có: Lời giải 2: Sử dụng bất đẳng thức AM-GM,ta có: Bất đẳng thức cuối đã rất quen thuộc,ta có đpcm. Ví dụ 3 : VMO 2002-Trần Nam Dũng Chứng minh rằng với mọi ,ta có: Lời giải: Sử dụng bất đẳng thức AM-GM,ta có: Mặt khác sử dụng bất đẳng thức Schur, Do đó Bất đẳng thức được chứng minh. Ví dụ 4 : Arqady Cho a,b,c là các số không âm,trong đó không có 2 số nào đồng thời bằng 0.Chứng minh rằng: Lời giải: Bất đẳng thức cần chứng minh tương đương với Áp dụng bất đẳng thức Cauchy-Schwarz,ta có: Áp dụng 2 bất đẳng thức trên,ta có: Giả sử và đặt . Ta cần chứng minh Bất đẳng thức cuối dễ dàng chứng minh bằng cách xét 2 trường hợp: và Đẳng thức xảy ra khi và . Ví dụ 5 : Moldova TST 2005 : Chứng minh rằng nếu a,b,c là các số thực dương và thì: Lời giải: Quy đồng mẫu số rồi khai triển,ta cần chứng minh: Áp dụng bất đẳng thức Schur và giả thiết ,ta có: Áp dụng BDT AM-GM,ta có: Mặt khác ta lại có: Vậy ta có đpcm. Ví dụ 6 : Vasile Cirtoaje : Cho là các số thực không âm thỏa mãn .Chứng minh rằng: Lời giải: Áp dụng BDT Schur,ta có: và Ta cần chứng minh: Bất đẳng thức cuối hiển nhiên đúng nên ta có đpcm. Đẳng thức xảy ra khi . Ví dụ 7 : Võ Thành Văn : Cho .Chứng minh rằng: Lời giải: Đổi biến theo p,q,r,bât đẳng thức cần chứng minh được viết lại như sau: ( ) Mặt khác,theo BDT Schur,ta có: Vậy ta có đpcm Ví dụ 8: Phạm Kim Hùng Cho a,b,c là các số thực dương thỏa mãn .Chứng minh rằng: Lời giải: Quy đồng,rút gọn và đổi biến theo p,q,r,bất đẳng thức cần chứng minh tương đương với Áp dụng BDT Schur,ta có: Từ giả thiết Thay 2 điều trên vào bất đẳng thức cần chứng minh,ta có: Bất đẳng thức cuối đúng nên ta có đpcm. Ví dụ 9: CRUX Cho a,b,c là các số thực không âm thỏa mãn .Chứng minh rằng: * Bài này đã được anh Hùng sử dụng cho phần BDT TRê-bư-sép trong cuốn Sáng tạo BDT,tuy nhiên bây giờ các bạn sẽ được thấy một lời giải với BDT Schur và phương pháp đổi biến p,q,r rất tự nhiên. Lời giải: Biến đổi bất đẳng thức cần chứng minh và chuyển về dạng ,ta có: Theo BDT AM-GM thì Theo BDT Schur,ta có: Nên ta cần chứng minh: Vậy BDT được chứng minh. 3.2: Phương pháp đổi biến p,q,r: Ví dụ 10 : Phạm Kim Hùng Cho a,b,c là các số thực không âm thỏa mãn a+b+c=3.Chứng minh rằng: Lời giải: Quy đồng mẫu số rồi khai triển,ta cần chứng minh: Sử dụng bất đẳng thức quen biết ,ta có: Tiếp tục sử dụng bất đẳng thức trên,ta cần chứng minh: hay với . Áp dụng BDT AM-GM,ta có nên cần chứng minh: Bắt đẳng thức cuối hiển nhiên đúng nên ta có đpcm. Đẳng thức xảy ra khi hoặc và các hoán vị. Ví dụ 11 : Dương Đức Lâm Cho a,b,c > 0.Chứng minh rằng Lời giải: Võ Quốc Bá Cẩn Đặt bất đẳng thức cần chứng minh tương đương với Áp dụng BDT Cauchy-Schwarz,ta có Đến đây ta cần chứng minh Giả sử chuyển về dạng ,BDT trở thành Từ ta có: Ví dụ 12 : Võ Quốc Bá Cẩn Cho thỏa mãn .Chứng minh rằng Lời giải: Nguyễn Thúc Vũ Hoàng Quy đồng,rút gọn,đổi biến BDT thành và xét hàm theo ,ta cần chứng minh Áp dụng BDT Schur,ta có .TH1: ,khi đó .TH2: ,khi đó với Đẳng thức xảy ra khi . Ví dụ 13: Vasc Cho a,b,c là các số thực không âm.Chứng minh rằng: Lời giải: Chuẩn hóa ,ta có bất đẳng thức: Đến đây ta sử dụng một thủ thuật khi dùng bất đẳng thức Schur ,đó là chia trường hợp để giải quyết: Nếu thì ta có và (dpcm) Nếu ta đưa bất đẳng thức cần chứng minh thành một hàm theo : Xét Vì nên suy ra Vậy bất đẳng thức được chứng minh. Đẳng thức xảy ra khi và các hoán vị *Với kĩ thuật xét trường hợp để giải ,chúng ta có thể dễ dàng giải quyết các bài toán sau: Bài toán 1: Xét 3 số thực không âm thỏa mãn .Chứng minh rằng: Gợi ý: Nhân vào rồi rút gọn,chuyển BDT về dạng p,q,r,ta cần chứng minh Đến đây chúng ta xét 2 TH và Bài toán 2: Cho a,b,c là các số thực không âm thỏa mãn abc=1.Chứng minh rằng Gợi ý:Đưa BDT về 1 hàm theo : Đến đây chúng ta chia thành 2 TH: TH1: TH2: vo thanh van Jun 3 2008, 08:37 PM Ví dụ 14 : Nguyễn Phi Hùng : Cho là các số thực không âm thỏa mãn .Chứng minh rằng: Lời giải: Theo giả thiết ta có Mặt khác theo bất đẳng thức Schur bậc 4 ta có: Vì vậy ta cần chứng minh (đpcm) Đẳng thức xảy ra và các hoán vị. Ví dụ 15:Cho và .Chứng minh rằng Lời giải: Đổi biến thành ,ta có bổ đề: Áp dụng BDT Cauchy-Schwarz,ta có: Ta có: Nên cần chứng minh Sử dụng bổ đề,ta có: vo thanh van Jun 3 2008, 08:38 PM Ví dụ 16 : Võ Thành Văn Cho các số thực dương a,b,c thỏa mãn .Chứng minh rằng: Lời giải: Áp dụng bất đẳng thức Schur,ta có: Bất đẳng thức cần chứng minh tương đương với: Bất đẳng thức trên hiển nhiên đúng theo BDT AM-GM nên ta có đpcm. Ví dụ 17 : Nguyễn Mạnh Dũng Cho . Chứng minh rằng: Lời giải: Ta có: Đặt Bất đẳng thức cần chứng minh trở thành Áp dụng BDT Schur,ta có: Ta cần chứng minh: Bất đẳng thức cuối hiển nhiên đúng vì và Ta có đpcm Đẳng thức xảy a khi và chỉ khi và các hoán vị vo thanh van Jun 3 2008, 08:40 PM Ví dụ 18 : HSG Toán QG THPT năm 2006 bảng B Chứng minh rằng với mọi số dương a,b,c thỏa mãn abc=1 thì Lời giải: Đặt ,ta có ,đồng thời đổi biến thành p,q,r,ta có bất đẳng thức: Mà bất đẳng thức trên đúng theo BDT Schur nên ta có đpcm. Ví dụ 19 : Phạm Sinh Tân Cho la các số thực không âm , tìm số nhỏ nhất để biểu thức sau đạt giá trị nhỏ nhất và đẳng thức xảy ra khi ba biến lệch nhau , chỉ rõ đằng thức xảy ra Lời giải: Đổi biến BDT theo p,q,r và chuẩn hóa p=1.Ta cần chứng minh bất đẳng thức: Ta có: Đẳng thức xảy ra khi hoặc và các hoán vị. *Một số bài tập tương tự: Bài toán 3 : Phạm Sinh Tân Với 3 số thực không âm a,b,c,chứng minh rằng Bài toán 4 : Phạm Sinh Tân Với a,b,c là các số thực không âm và không có quá hai số nào đồng thời bằng 0.Chứng minh rằng: vo thanh van Jun 3 2008, 08:42 PM Ví dụ 20 : Dương Đức Lâm Cho a,b,c là các số thực không âm.Chứng minh rằng: Lời giải: Ta có: Đặt ,ta có: Lúc đó BDT trở thành Đưa bất đẳng thức về dạng ,từ giả thiết,ta có và lúc đó,bất đẳng thức trở thành Nếu ,sử dụng BDT Schur,ta có: điều này đúng vì Nếu ,ta có , Vậy BDT được chứng minh Đẳng thức xảy ra khi hoặc và các hoán vị vo thanh van Jun 3 2008, 08:44 PM Ví dụ 21: Cho a,b,c là các số thực không âm thỏa mãn a+b+c=3.Chứng minh rằng: Lời giải: Chuyển đổi BDT về như sau: Ta thấy BDT trên đúng do : Theo BDT AM-GM : và theo BDT Schur thì Vậy BDT đựoc chứng minh. Đẳng thức xảy ra và các hoán vị Ví dụ 22 : Darij Grinberg Cho là các số thực không âm và không có hai số nào đồng thời bằng 0.Chứng minh rằng: Lời giải: Áp dụng BDT Cauchy-Schwarz,ta cần chứng minh: Đổi biến theo p,q,r,khi đó bất đẳng thức viết thành: Áp dụng BDT Schur,ta có và BDT quen biết ,ta có đpcm. Ví dụ 23: Muhai Piticari,Dan Popescu Chứng minh rằng với và ,ta có Lời giải: Đổi biến về p,q,r,ta cần chứng minh Mặc khác BDT trên đúng theo BDT Schur nên ta có đpcm. Và một ví dụ điển hình cho phương pháp này là BDT IRAN 96: Ví dụ 24 : IRAN 96 : Chứng minh rằng nếu thì Lời giải: Sử dụng phương pháp đổi biến p,q,r,ta chuyển bất đẳng thức về dạng như sau: Biến đổi tương đương,rút gọn,ta cần chứng minh Bất đẳng thức cuối đúng nên ta có đpcm. vo thanh van Jun 3 2008, 08:47 PM Qua các ví dụ trên,có lẽ các bạn cũng đã hình dung được ít nhiều về bất đẳng thức Schur và những ứng dụng của nó trong phương pháp đổi biến p,q,r.Để kết thúc bài viết này,mời các bạn cùng giải một số bài tập sau: Bài tập 1:Cho a,b,c là các số thực không âm thoả mãn .Chứng minh rằng (Vasile Cirtoaje) Bài tập 2: Darij Grinberg Cho a,b,c là các số thực không âm.Chứng minh rằng Bài tập 3: Vasile Cirtoaje Cho a,b,c là các số thực không âm thoả mãn Chứng minh rằng Bài tập 4:Vũ Đình Quý Cho 3 số thực dương thỏa mãn abc=1.Chứng minh rằng: Bài tập 5:VMO 2002 Chứng minh rằng với các số thực a,b,c thoả mãn ,ta có Bài tập 6: Junior TST 2003,Romania Cho a,b,c là các số thực dương thoả mãn .Chứng minh rằng: Bài tập 7: Balkan Contest Cho là các số thực dương thoả mãn .Chứng minh rằng: . triển bất đẳng thức trên,ta cần chứng minh: Ta có: (theo BDT Schur) Áp dụng các BDT trên,ta có: Lời giải 2: Sử dụng bất đẳng thức AM-GM,ta có: Bất đẳng thức. rằng với mọi ,ta có: Lời giải: Sử dụng bất đẳng thức AM-GM,ta có: Mặt khác sử dụng bất đẳng thức Schur, Do đó Bất đẳng thức được chứng minh. Ví dụ 4 : Arqady

Ngày đăng: 20/07/2013, 01:25

Từ khóa liên quan

Tài liệu cùng người dùng

  • Đang cập nhật ...

Tài liệu liên quan